WBR0392

Revision as of 05:53, 21 November 2013 by Rim Halaby (talk | contribs) (Created page with "{{WBRQuestion |QuestionAuthor={{Rim}} |ExamType=USMLE Step 1 |MainCategory=Pharmacology |SubCategory=Infectious Disease |MainCategory=Pharmacology |SubCategory=Infectious Dise...")
(diff) ← Older revision | Latest revision (diff) | Newer revision → (diff)
Jump to navigation Jump to search
 
Author [[PageAuthor::Rim Halaby, M.D. [1]]]
Exam Type ExamType::USMLE Step 1
Main Category MainCategory::Pharmacology
Sub Category SubCategory::Infectious Disease
Prompt [[Prompt::A 35 year old man previously diagnosed with HIV presents to the infectious disease clinic at a local hospital for a regular follow-up. Although he was not treated, his last CD4 count 3 months ago was 670 cells/mm3 and his viral load was low to undetectable. In the past month, the patient was admitted twice to the hospital for a community acquired pneumonia and a gastroenteritis. His new tests show a CD4 of 231 cells/mm3 and a viral load of 325000 copies/ml. The infectious disease physician decides to initiate highly active antiretroviral therapy (HAART) in this patient given the recent findings. Which of the following drug regimens are suitable?]]
Answer A AnswerA::Tenofovir + emtricitabine + abacavir
Answer A Explanation AnswerAExp::
Answer B AnswerB::Darunavir + raltegravir + Lamivudine
Answer B Explanation AnswerBExp::
Answer C AnswerC::Efavirenz + tenofovir + nevirapine
Answer C Explanation AnswerCExp::
Answer D AnswerD::Ritonavir + tenofovir + lopinavir
Answer D Explanation AnswerDExp::
Answer E AnswerE::Raltegravir + tenofovir + emtricitabine
Answer E Explanation AnswerEExp::
Right Answer RightAnswer::E
Explanation [[Explanation::

Educational Objective:
References: ]]

Approved Approved::No
Keyword WBRKeyword::HAART therapy, WBRKeyword::HIV, WBRKeyword::AIDS, WBRKeyword::CD4 count, WBRKeyword::HIV drug combinations
Linked Question Linked::
Order in Linked Questions LinkedOrder::